Meine Merkliste
my.chemie.de  
Login  

Ehrenfest-Theorem



Das Ehrenfest-Theorem, benannt nach dem österreichischen Physiker Paul Ehrenfest, stellt innerhalb der Physik einen Zusammenhang zwischen der klassischen Mechanik und der Quantenmechanik her. Es besagt, dass unter bestimmten Bedingungen die klassischen Bewegungsgleichungen für die Mittelwerte der Quantenmechanik gelten; die klassische Mechanik also in gewissem Maße in der Quantenmechanik enthalten ist (Korrespondenzprinzip).

Mathematisch drückt sich das in seiner allgemeinsten Form so aus, dass die vollständige Zeitableitung des Erwartungswertes eines quantenmechanischen Operators mit dem Kommutator dieses Operators und des Hamiltonoperators wie folgt in Zusammenhang stehen:

\frac{d}{dt}\langle O\rangle = \frac{i}{\hbar}\langle [H,O] \rangle + \left\langle \frac{\partial O}{\partial t}\right\rangle

Dabei stellt O einen quantenmechanischen Operator und \langle O \rangle dessen Erwartungswert dar.

Inhaltsverzeichnis

Anwendung

Im Hamilton-Formalismus der klassischen Mechanik gilt für die Zeitentwicklung einer Phasenraumfunktion:

\frac{d}{dt}f(p,q,t) = \{H,f\} + \frac{\partial f}{\partial t}

mit der Poisson-Klammer \{H,f\} = \nabla_q H \nabla_p f - \nabla_p H \nabla_q f. Bei der Quantisierung wird die Poisson-Klammer durch den mit i\hbar multiplizierten Kommutator ersetzt. Das quantenmechanische Analogon einer Phasenraumfunktion ist ein Operator (Observable). Somit ist das Ehrenfest-Theorem das direkte Analogon zu der obigen klassischen Aussage.

Für den Spezialfall des nicht explizit zeitabhängigen Impulsoperators p gilt somit:

\frac{d}{dt} \langle p \rangle = \frac{i}{\hbar} \langle [H,p] \rangle = \frac{i}{\hbar} \langle [V,p]\rangle = \frac{i}{\hbar} \langle -\frac{\hbar}{i} \nabla V \rangle = - \langle \nabla V \rangle

Da weiterhin für die Zeitentwicklung des Ortsoperators folgt

\frac{d}{dt}\langle x\rangle = \frac{i}{\hbar}\langle[H,x]\rangle =\frac{1}{m}\langle p\rangle

lässt sich folgern:

m\frac{d^2}{dt^2}\langle x\rangle = -\langle\nabla V\rangle = \langle F(x)\rangle

Lässt sich nun der Erwartungswert der Funktion F der Position x durch die Funktion F des Erwartungswerts der Position x nähern, so erhält man

\langle F(x)\rangle\approx F(\langle x \rangle) \qquad (*)

und somit

m\frac{d^2}{dt^2}\langle x\rangle = F(\langle x\rangle).

In Worten bedeutet dies, dass sich das Maximum der Aufenthaltswahrscheinlichkeit auf einer klassischen Bahn bewegt, d.h. der klassischen Bewegungsgleichung folgt. Das Ehrenfest-Theorem führt somit direkt auf eine Analogie der Quantenmechanik zur klassischen Mechanik - hier in Form des zweiten Newton'schen Axioms

m\frac{d^2}{dt^2}x=ma(x)=F(x).

Die Annahme (*) und damit auch die klassische Bewegungsgleichung für quantenmechanische Erwartungswerte gelten allerdings nur, falls die Kraft F(x) eine lineare Funktion der Position x ist. Dies gilt für die einfachen Fälle des harmonischen Oszillators oder des freien Teilchens. Außerdem kann man sagen, dass (*) gilt, wenn die Breite der Aufenthaltswahrscheinlichkeit klein ist gegenüber der typischen Längenskala auf der die Kraft F(x) variiert.

Herleitung

Es sei das betrachtete System im Quantenzustand Ψ. Man erhält somit für die Zeitableitung des Erwartungswertes eines Operators O:

\frac{d}{dt}\langle O \rangle = \frac{d}{dt}\int \Psi^* O \Psi dV = \int \left(\frac{\partial \Psi^*}{\partial t}\right)O\Psi + \Psi^*\left(\frac{\partial O}{\partial t}\right)\Psi + \Psi^* O \left(\frac{\partial \Psi}{\partial t}\right)dV =\int \left(\frac{\partial \Psi^*}{\partial t}\right) O \Psi + \Psi^* O \left(\frac{\partial \Psi}{\partial t}\right)dV + \langle \frac{\partial O}{\partial t}\rangle

Man betrachtet nun die Schrödingergleichung

\frac{\partial \Psi}{\partial t} = \frac{1}{i\hbar}H\Psi

\frac{\partial \Psi^*}{\partial t} = \frac{-1}{i\hbar}\Psi^*H^*=\frac{-1}{i\hbar}\Psi^*H

Im zweiten Schritt wurde mit H = H * ausgenutzt, dass der Hamilton-Operator selbstadjungiert ist. Einsetzen dieser Relationen liefert nun:

\frac{d}{dt}\langle O\rangle = \frac{1}{i\hbar}\int -\Psi^* HO \Psi + \Psi^* OH\Psi dV + \langle\frac{\partial O}{\partial t}\rangle=\frac{1}{i\hbar}\langle[O,H]\rangle + \langle \frac{\partial O}{\partial t}\rangle = \frac{i}{\hbar}\langle[H,O]\rangle + \langle \frac{\partial O}{\partial t}\rangle

Siehe auch

  • Liouville-Gleichung, Satz von Liouville (Physik)

Literatur

  • Leslie E. Ballentine: Quantum Mechanics: A Modern Development 1. Auflage. World Scientific Publishing, Singapore 1998, ISBN 981-02-4105-4
  • P. Ehrenfest: Bemerkung über die angenäherte Gültigkeit der klassischen Mechanik innerhalb der Quantenmechanik Zeitschrift für Physik A Ausgabe 45, Nummern 7-8 / Juli, 1927, Seiten 455-457.
 
Dieser Artikel basiert auf dem Artikel Ehrenfest-Theorem aus der freien Enzyklopädie Wikipedia und steht unter der GNU-Lizenz für freie Dokumentation. In der Wikipedia ist eine Liste der Autoren verfügbar.
Ihr Bowser ist nicht aktuell. Microsoft Internet Explorer 6.0 unterstützt einige Funktionen auf ie.DE nicht.